LSAT and Law School Admissions Forum

Get expert LSAT preparation and law school admissions advice from PowerScore Test Preparation.

 srcline@noctrl.edu
  • Posts: 243
  • Joined: Oct 16, 2015
|
#27689
Hello

So I am a bit confused on what this question stem is asking. It states "the facts described above provide the strongest evidence against which one of the following?

So since this stimulus says "facts" there's no conclusion but its not a must be true b/c it saying "evidence against which one of the following? Is this a weaken then? Your trying to find a answer choice that is unsupported by the stimulus correct....so a must be false like in logic games?

Thankyou
Sarah
 Claire Horan
PowerScore Staff
  • PowerScore Staff
  • Posts: 408
  • Joined: Apr 18, 2016
|
#27715
Hi Sarah,

You are right--this is a must be false/cannot be true question.

The question stem reads: "The facts described above provide the strongest evidence against which one of the following?"

In this question, "strongest evidence against" is essentially asking, based on the facts, "Which of the following is least likely to be true?" First, I would look for an answer choice that can't be true. It is possible that none of the answer choices are completely disproven, though. Maybe the stimulus just provides strong evidence or weakens one of the answer choices. So if you can't find one that "cannot be true," you can look for the one that is least likely to be true.

I hope this explanation was helpful!

-Claire
 Pragmatism
  • Posts: 68
  • Joined: Jan 11, 2018
|
#43291
For this Must be False question, this was my thought process for choosing "D" as my answer choice.

— A) This answer could be true, because the stimulus says, "finish work each day by setting a daily income target."

— B) Even though this is the correct answer choice, I figured, well if "people work longer when their effective hourly wage is high than when it is low," goes inline with setting a daily income target for themselves. So, if someone's income is does receive a "high wage," doesn't necessarily mean they work shorter hours, because their daily income target could be higher, and on those basis, I concluded, this could be true.

— C) could be true.

— D) Also, could be true, but the based on the fact that "they stop when they reach that target." Also, the conclusion supports this now that I am typing this out, so shame on me.

— E) is out of scope.

My question is, while ALL answer choice could be except for E, which is out of scope, is B correct for the simple reason that, the stimulus gives less support to that could be true answer than other answers that are either could be true or must be true?
 Claire Horan
PowerScore Staff
  • PowerScore Staff
  • Posts: 408
  • Joined: Apr 18, 2016
|
#43686
Hi Pragmatism,

The fact given in the stimulus, that people "typically work fewer hours on a busy day than a slow day" directly contradicts answer choice B and, thus, that is the right answer. I don't really follow the reasoning in your post for answer choice (B), but it's not a good idea to start with the answer choice and reason backwards to the stimulus on a Cannot Be True question.

Instead, you should start with the stimulus itself and prephrase. It's a cannot be true question, and the sentence I quoted above, plus the rest of the stimulus leads to the conclusion that on a busy day, when drivers are getting more per hour, they reach their income target sooner and thus end up working fewer hours that day. With this prephrase of what must be true, look for its logical opposite (its negative), which will be a statement that cannot be true. Working longer when people make more per hour, as answer choice B describes, is the opposite of the prephrase we deduced from the information in the stimulus.

I hope this helps!

Get the most out of your LSAT Prep Plus subscription.

Analyze and track your performance with our Testing and Analytics Package.